Last visit was: 31 Aug 2024, 18:53 It is currently 31 Aug 2024, 18:53
Close
GMAT Club Daily Prep
Thank you for using the timer - this advanced tool can estimate your performance and suggest more practice questions. We have subscribed you to Daily Prep Questions via email.

Customized
for You

we will pick new questions that match your level based on your Timer History

Track
Your Progress

every week, we’ll send you an estimated GMAT score based on your performance

Practice
Pays

we will pick new questions that match your level based on your Timer History
Not interested in getting valuable practice questions and articles delivered to your email? No problem, unsubscribe here.
Close
Request Expert Reply
Confirm Cancel
SORT BY:
Kudos
Tags:
Show Tags
Hide Tags
Retired Moderator
Joined: 05 May 2019
Status:GMAT Club Team member
Affiliations: GMAT Club
Posts: 997
Own Kudos [?]: 829 [1]
Given Kudos: 1005
Location: India
GMAT Focus 1:
645 Q82 V81 DI82
GMAT 1: 430 Q31 V19
GMAT 2: 570 Q44 V25
GMAT 3: 660 Q48 V33
GPA: 3.26
WE:Engineering (Manufacturing)
Send PM
Retired Moderator
Joined: 05 May 2019
Status:GMAT Club Team member
Affiliations: GMAT Club
Posts: 997
Own Kudos [?]: 829 [1]
Given Kudos: 1005
Location: India
GMAT Focus 1:
645 Q82 V81 DI82
GMAT 1: 430 Q31 V19
GMAT 2: 570 Q44 V25
GMAT 3: 660 Q48 V33
GPA: 3.26
WE:Engineering (Manufacturing)
Send PM
Retired Moderator
Joined: 05 May 2019
Status:GMAT Club Team member
Affiliations: GMAT Club
Posts: 997
Own Kudos [?]: 829 [1]
Given Kudos: 1005
Location: India
GMAT Focus 1:
645 Q82 V81 DI82
GMAT 1: 430 Q31 V19
GMAT 2: 570 Q44 V25
GMAT 3: 660 Q48 V33
GPA: 3.26
WE:Engineering (Manufacturing)
Send PM
Retired Moderator
Joined: 05 May 2019
Status:GMAT Club Team member
Affiliations: GMAT Club
Posts: 997
Own Kudos [?]: 829 [1]
Given Kudos: 1005
Location: India
GMAT Focus 1:
645 Q82 V81 DI82
GMAT 1: 430 Q31 V19
GMAT 2: 570 Q44 V25
GMAT 3: 660 Q48 V33
GPA: 3.26
WE:Engineering (Manufacturing)
Send PM
Verbal Question of the Day Chat Group [#permalink]
1
Kudos
Retired Moderator
Joined: 05 May 2019
Status:GMAT Club Team member
Affiliations: GMAT Club
Posts: 997
Own Kudos [?]: 829 [1]
Given Kudos: 1005
Location: India
GMAT Focus 1:
645 Q82 V81 DI82
GMAT 1: 430 Q31 V19
GMAT 2: 570 Q44 V25
GMAT 3: 660 Q48 V33
GPA: 3.26
WE:Engineering (Manufacturing)
Send PM
Re: Verbal Question of the Day Chat Group [#permalink]
1
Kudos
Retired Moderator
Joined: 05 May 2019
Status:GMAT Club Team member
Affiliations: GMAT Club
Posts: 997
Own Kudos [?]: 829 [1]
Given Kudos: 1005
Location: India
GMAT Focus 1:
645 Q82 V81 DI82
GMAT 1: 430 Q31 V19
GMAT 2: 570 Q44 V25
GMAT 3: 660 Q48 V33
GPA: 3.26
WE:Engineering (Manufacturing)
Send PM
Re: Verbal Question of the Day Chat Group [#permalink]
1
Kudos
Intern
Intern
Joined: 26 Jul 2023
Posts: 4
Own Kudos [?]: 1 [1]
Given Kudos: 17
Send PM
Re: Verbal Question of the Day Chat Group [#permalink]
1
Kudos
I have a question from focus mock, can anyone help me with it

Jacob: Public funding of the arts is worthwhile for our city because publicly funded art makes the city more attractive to new residents and businesses, thus enhancing the city’s tax base.
Andrew: That argument is misguided. Art’s true value lies in being a profound expression of human nature. Funding art solely in order to reach economic goals debases it by disregarding its intrinsic value.
Based on their statements, Jacob and Andrew most clearly disagree about whether
• a sufficient rationale for public funding of the arts in the city is the potential effects on the city’s tax base
• publicly funding the arts in the city would make the city more attractive to new residents and businesses
• public funding of the arts is likely to be economically worthwhile for the city
• publicly funding the arts in the city would lower the artistic quality of the arts there
• funding the arts while respecting art’s intrinsic value necessarily entails ignoring issues concerning what monetary costs are reasonable

this is from focus mock 3
Manager
Manager
Joined: 05 May 2018
Posts: 53
Own Kudos [?]: 72 [1]
Given Kudos: 9
Send PM
Re: Verbal Question of the Day Chat Group [#permalink]
1
Kudos
I like B and E for today’s sentence correction questions
Verbal Chat Moderator
Joined: 20 Mar 2018
Posts: 1948
Own Kudos [?]: 1630 [1]
Given Kudos: 1680
Send PM
Re: Verbal Question of the Day Chat Group [#permalink]
1
Kudos
poojaarora1818 wrote:
Are these butler ques need to be addressed in their respective threads only?

you can post your solution in question thread and get a kudos. Also your explanation could be helpful for others or you can discuss with others about solution
Math Expert
Joined: 02 Sep 2009
Posts: 95291
Own Kudos [?]: 654402 [1]
Given Kudos: 87117
Send PM
Re: Verbal Question of the Day Chat Group [#permalink]
1
Kudos
Expert Reply
AbhinavShah14 wrote:
Bunuel wrote:
Critical Reasoning Butler: January 2024
January 3CR 1CR 2

When will the Answers be revealed for them?


Check the note in the fist post:

"The OA will be automatically revealed on Wednesday 3rd of January 2024 01:30:13 PM Pacific Time Zone".

Attachment:
2024-01-03_16-56-08.png
2024-01-03_16-56-08.png [ 76.44 KiB | Viewed 353 times ]
Verbal Chat Moderator
Joined: 20 Mar 2018
Posts: 1948
Own Kudos [?]: 1630 [1]
Given Kudos: 1680
Send PM
Re: Verbal Question of the Day Chat Group [#permalink]
1
Kudos
winterschool wrote:
Q1. Solo concert pianists, by convention, are not permitted to use musical scores during their performances. However, most members of chamber groups and orchestras are permitted to use sheet music during performances and perform well as a result. Therefore, all solo concert pianists should also be allowed to consult their musical scores during performances. The argument is most vulnerable to criticism on which of these grounds? (A) It overlooks the possibility that some solo concert pianists prefer performing without consulting musical scores. (B) It takes for granted that members of a chamber group or orchestra are less skilled than solo musicians and thus have more need for musical scores. (C)It overlooks the possibility that some solo concert pianists have broken with tradition and used musical scores during their performances. (D) It takes for granted that a solo concert pianist would use a musical score in the same way as does a member of a chamber group or orchestra. (E) It overlooks the possibility that performing in an orchestra is difficult despite the ability to use a musical score during the performance.



winterschool wrote:
Q2. Manager: Our company’s mail-order sales have recently increased 25 percent. This increase started around the time we started offering unlimited free shipping, rather than just free shipping on orders over $50. Thus, our change in policy probably caused the increase. Which one of the following, if true, most strengthens the manager’s argument? (A) Mail-order sales have been decreasing for companies that do not offer unlimited free shipping. (B) The company did not widely advertise its change in policy. (C) The company’s profits from mail-order sales have increased since the change in policy. (D) The company’s change in policy occurred well after its competitors started offering unlimited free shipping. (E) Most companies offer free shipping only on mail-order purchases over $50. Difficulty - Hard



CR Questions January - 12 :

Q1. The paintings of French painter Trianne Dejere sold best in the period following the production of La Triump now Dejere’s most famous piece. In the 12-month period preceding the unveiling of this piece, Dejere sold 57% of the works she produced in this period, a far greater percentage than in previous years. In the 12-month period following a glowing review of La Triumph in a popular magazine, however, Dejere sold 85% of the paintings she produced. Interestingly, Dejere’s revenue from painting sales was roughly the same in both periods, since she sold the same number of paintings in the 12 months before presenting La Triumph as she did in the 12 months following the favorable review.

Which of the following statements can be properly concluded from the passage, if the information above is true?


(A) Due to the positive review, Dejere was able to charge substantially more for the works produced after La Triumph than the works produced before it.

(B) Dejere was more concerned with positive reviews than with increasing the prices of her paintings.

(C) The positive review of La Triumph brought Dejere’s work to the attention of more art collectors than were previously aware of her work.

(D) Dejere painted fewer works in the 12-month period following the review of La Triumph than she had in the 12-month period preceding its unveiling.

(E) Dejere paid more attention to marketing her paintings after La Triumph received such a positive reception

Q2. Because of the rising costs of gasoline, a small bus company has decided to decrease the number of weekend trips it runs to a nearby metropolitan area by one third. It will not lose any riders, because the decreased number of trips is adequate for the existing number of weekend riders. The company will save substantially on fuel costs and will have one fewer driver working on weekends. Moreover, the bus company will continue to raise revenue through print advertising inside the bus and on the bus ’ s exterior.

Which of the following if true, provides the strongest evidence that the bus company ’ s profits are likely to decrease if the company implements the plan?

a) Many of the weekend riders would continue riding the bus even if the fare was increased.

b)Most of the advertisers will continue to spend the same amount on advertising per mile and the displaced weekend driver will not earn more by switching to a weekday route at the company.

c) The displaced weekend driver will earn the same salary driving on weekdays and the advertisers will increase their advertising on the weekend.

d) Maintenance costs per bus will remain constant.

e) The increased cost of fuel increases the operating costs of one round trip by twenty per cent.



Difficulty - Hard
Manager
Manager
Joined: 25 Nov 2022
Posts: 203
Own Kudos [?]: 113 [1]
Given Kudos: 41
Location: India
Concentration: Entrepreneurship, General Management
GMAT 1: 660 Q47 V34
GMAT 2: 640 Q48 V29
GPA: 4
WE:General Management (Manufacturing)
Send PM
Re: Verbal Question of the Day Chat Group [#permalink]
1
Kudos
Bunuel wrote:
Critical Reasoning Butler: January 2024
January 24CR 1CR 2

CR1: E CR2: B - Both looks to be in the "Easy" category. But fun to solve.
Founder
Founder
Joined: 04 Dec 2002
Posts: 38342
Own Kudos [?]: 74987 [1]
Given Kudos: 20856
Location: United States (WA)
GMAT 1: 750 Q49 V42
GPA: 3
Send PM
Re: Verbal Question of the Day Chat Group [#permalink]
1
Kudos
Expert Reply
winterschool wrote:
could be mixed or new set of questions. But possibility of mixed questions is greater

I should have an answer. You can reset the free tests unlimited number of times but you should be able to get at least two unique tests out of every single free one. Potentially three without any repeats. You can obviously reset them as many times as you want.

For the paid tests, supposedly they can be reset once and you can get two unique tests out of each. You can only have one reset for the paid tests.
Founder
Founder
Joined: 04 Dec 2002
Posts: 38342
Own Kudos [?]: 74987 [1]
Given Kudos: 20856
Location: United States (WA)
GMAT 1: 750 Q49 V42
GPA: 3
Send PM
Re: Verbal Question of the Day Chat Group [#permalink]
1
Kudos
Expert Reply
JRutuja wrote:
How is the difficult level of next questions affected when the previous question is marked for review? If I mark all 3 MSR questions for review and come back to those at the end... will this reduce the overall difficulty level of my section?

It’s a good question. Unfortunately we don’t know yet. There are several ways to influence an adaptive algorithm in a test where and choices can be changed and similarly thorough ways to counteract these hacks. The reality however is that because the test database is limited, even if you get an arithmetic question right in quant and your next question should be harder algebra question but the database has only easy algebra questions, you’ll get an easy question similarly if you get the question wrong and you’re supposed to get an easy probability question, but the database only has hard probability questions, you’ll get a hard probability question so there is variance in question difficulty simply because of the database size. Back to your question, GMAC claims that supposedly the three and choices don’t impact the final score but who knows for sure…. Ultimately we haven’t been able to get enough information about the algorithm to determine or to get a good understanding of it like we had in the version of the test.

Personally, I wouldn’t worry too much about how the difficulties determined and focus on how you can get as many questions answered correctly as possible.
Manager
Manager
Joined: 24 Oct 2023
Posts: 103
Own Kudos [?]: 14 [1]
Given Kudos: 132
Location: India
Send PM
Re: Verbal Question of the Day Chat Group [#permalink]
1
Kudos
Difficulty levels have been updated to match the focus edition scores. Some are missing. Not an important feature, just telling. e.g. critical-reasoning-question-directory-topic-and-difficulty-128861.html#p1056423
Intern
Intern
Joined: 17 Apr 2024
Posts: 3
Own Kudos [?]: 3 [1]
Given Kudos: 4
Send PM
Re: Verbal Question of the Day Chat Group [#permalink]
1
Kudos
The question asks which of the statements furthers the conclusion given in the question. In C , if Vernia’s output is less than Sylvania, then it is possible that Vernia could produce less output than Borodia as we are not given any evidence of Vernia’s output being more than Borodia’s
Manager
Manager
Joined: 05 Mar 2024
Posts: 156
Own Kudos [?]: 31 [1]
Given Kudos: 7
Send PM
Re: Verbal Question of the Day Chat Group [#permalink]
1
Kudos
Hello.
I got a question.

Mike has listened to songs a total number of 870 hours after getting his new phone. How many days is required for him, listening 10 hours per day to reach that 870 hours of play time?
Manager
Manager
Joined: 19 Feb 2022
Status:Preparing for the GMAT
Posts: 115
Own Kudos [?]: 34 [1]
Given Kudos: 62
Location: India
GMAT 1: 700 Q49 V35
GPA: 3.33
WE:Consulting (Consulting)
Send PM
Re: Verbal Question of the Day Chat Group [#permalink]
1
Kudos
Vishnu0001 wrote:
It is funny because the questions posted here are easy. I did the FREE mock 1 by GMAC and the questions are supertough. Now I’m in the search of a question bank that could train me properly...any suggestions?

GMAT OG, OG Review, OG Advanced. If all this isn’t enough, please move on to LSAT CR and RCs. They are tough, but mastering them makes the GMAT verbal a cakewalk.
Founder
Founder
Joined: 04 Dec 2002
Posts: 38342
Own Kudos [?]: 74987 [1]
Given Kudos: 20856
Location: United States (WA)
GMAT 1: 750 Q49 V42
GPA: 3
Send PM
Re: Verbal Question of the Day Chat Group [#permalink]
1
Kudos
Expert Reply
Jithumathai wrote:
Could anyone advise what is the main idea behind "assumption " On CR Questions

There are some topics on assumptions written quite well. Assumptions are always unstated. So it is not a repeat or conclusion. An assumption is an invisible bridge between the statements and the conclusion. If you break the assumption, the bridge is gone and conclusion will not work. Finding assumptions is actually a fun exercise.
Manager
Manager
Joined: 06 Jun 2024
Posts: 234
Own Kudos [?]: 249 [1]
Given Kudos: 33
Send PM
Verbal Question of the Day Chat Group [#permalink]
1
Kudos
Expert Reply
 
Nullbyte wrote:
But we don’t know if hormone X is a cause of tremor or not. We are assuming that just because they are present together

For example if you say "yesterday Henry drank cold water and he got sick today so cold water must have caused the sickness".
But here we are assuming that nothing else caused the sickness

We are not. This assumption is specious because it is possible that drinking cold water led to 'infection X' and it is 'infection X' that caused the sickness. In other words, 'infection X' induces sickness, and 'infection X' could get triggered by various causes, one of the causes being drinking cold water. The conclusion in the question I have shared is this: When hormone X exceeds normal concentrations, tremors are induced. So, hormone X's exceeding normal concentrations could lead to a hormonal imbalance, and another hormone, hormone Y, may get impacted, leading to tremors. So, my question to you is this: Why should no other hormone other than X that causes tremors be an assumption?­
GMAT Club Bot
Verbal Question of the Day Chat Group [#permalink]
   1   2   3   4  ...  204   
Moderators:
GMAT Club Verbal Expert
7048 posts
GMAT Club Verbal Expert
234 posts
GRE Forum Moderator
14003 posts